LSAT and Law School Admissions Forum

Get expert LSAT preparation and law school admissions advice from PowerScore Test Preparation.

User avatar
 Dave Killoran
PowerScore Staff
  • PowerScore Staff
  • Posts: 5853
  • Joined: Mar 25, 2011
|
#88994
Complete Question Explanation
(The complete setup for this game can be found here: lsat/viewtopic.php?f=237&p=88987)

The correct answer choice is (D)

The only scenario that meets the condition in the question stem is as follows:

G4-Q19-d1.png

Of course, because of the second rule, K cannot sit next to H, leaving only M or P to sit next to H:

G4-Q19-d2.png

Answer choice (A): This answer choice cannot occur because K sits immediately next to F in this scenario.

Answer choice (B): This answer choice cannot occur because O already sits between G and I.

Answer choice (C): This answer choice cannot occur because H and I are too far apart.

Answer choice (D): This is the correct answer choice.

Answer choice (E): This answer choice cannot occur because while P could sit next to one of K and M, P cannot sit between both.
You do not have the required permissions to view the files attached to this post.

Get the most out of your LSAT Prep Plus subscription.

Analyze and track your performance with our Testing and Analytics Package.